nmop_apisdn2
Thanks Received: 16
Vinny Gambini
Vinny Gambini
 
Posts: 24
Joined: June 23rd, 2012
 
 
 

Q9 - It can be safely concluded that

by nmop_apisdn2 Wed Jul 11, 2012 9:14 pm

Okay, so the stimulus is not very tricky; its the answer choices that you must get through in order to find the correct answer.

(A) is incorrect because this is not sufficient to establish the conclusion and I'll give you some numbers to show why:

Lets assume that Martown has 100 trees and Seclee has 50 trees.

If more trees were planted in Seclee in the past two years than were in Martown, this doesn't prove the conclusion because if Seclee planted 10 trees the first you and 10 trees the second year, Seclee would only have 70 trees in total. Martown could have planted 0 trees and would still have more trees than Seclee. It is for these reasons that answer choice (A) is not sufficient and thus incorrect.

(C) is incorrect because even if Martown is suffering from an epidemic, this does not show that Seclee has at least as many trees. For example, if Martown has 100 trees and Seclee has 40 trees, and the epidemic killed off 50 of the trees in Martown, then Martown would still have more trees.

(D) is incorrect because the rainfall in the areas does not give us enough of a reason to believe that Seclee has at least as many trees as Martown.
(E) is incorrect because even if the average number of trees cut down annually in Martown is higher than in Seclee, Seclee could be cutting down 5 trees a year and Martown could be cutting down 6 trees a year, which would mean that this answer choice still holds true. However, as you can see, this would still not prove that Seclee has at least as many trees as Martown.

Finally, (B) is the correct choice because if Seclee is the region within Martown is located, then it must be true that there are at least as many trees in Seclee as in Martwon. Therefore, we have found the sufficient assumption.
User avatar
 
ohthatpatrick
Thanks Received: 3808
Atticus Finch
Atticus Finch
 
Posts: 4661
Joined: April 01st, 2011
 
This post thanked 1 time.
 
 

Re: Q9 - It can be safely concluded that

by ohthatpatrick Wed Jul 18, 2012 2:09 am

Awesome work!

Just to give (B) another spin for anyone confused, let's say that "Manhattan is the region within which Central Park is located."

Thus, there are at least as many trees in Manhattan as there are in Central Park.

After all, every tree in Central Park ALSO counts as a tree in Manhattan.
 
timsportschuetz
Thanks Received: 46
Elle Woods
Elle Woods
 
Posts: 95
Joined: June 30th, 2013
 
 
trophy
First Responder
 

Re: Q9 - It can be safely concluded that

by timsportschuetz Sun Jun 30, 2013 4:52 pm

I am a little bit confused... Above, you provide an example with specific numbers of trees. The way I am interpreting the premise, it follows that Seclee must have 'at least' as many trees as Martown. This would effectively mean that Seclee must always have an equal number or more trees than Martown.

Thus, your example of Martown having more trees than Seclee would be inaccurate...

Please help... am I reading the premise incorrectly?
User avatar
 
ohthatpatrick
Thanks Received: 3808
Atticus Finch
Atticus Finch
 
Posts: 4661
Joined: April 01st, 2011
 
 
 

Re: Q9 - It can be safely concluded that

by ohthatpatrick Sun Jun 30, 2013 8:09 pm

I think you're just getting turned around (understandably) on this really weirdly organized problem.

The stimulus is NOT a premise (as you said) ... it's the conclusion / the inference we're hoping to make.

You should never accept a conclusion as true; you should actively debate whether it needs to be true.

The correct answer to this question will give us a fact that PROVES the conclusion is true.

The incorrect answers will FAIL to prove the conclusion is true.

The original poster was demonstrating this by showing how some incorrect answer choices were compatible with M being greater than S.

(B), by contrast, is NOT compatible with M > S.

M = 100 and S = 50 contradicts what (B) is saying.

If M = 100, then S = at least 100, since all of M is inside S.

So that's why (B) is correct.

For all the other answer choices, M = 100 and S = 50 does NOT contradict the answer choice.

Hope this helps.
User avatar
 
WaltGrace1983
Thanks Received: 207
Atticus Finch
Atticus Finch
 
Posts: 837
Joined: March 30th, 2013
 
 
trophy
Most Thanked
trophy
Most Thankful
trophy
First Responder
 

Re: Q9 - It can be safely concluded that

by WaltGrace1983 Thu Jan 16, 2014 3:39 pm

Is this a really rare type of question? I have never seen anything like it.
User avatar
 
ohthatpatrick
Thanks Received: 3808
Atticus Finch
Atticus Finch
 
Posts: 4661
Joined: April 01st, 2011
 
 
 

Re: Q9 - It can be safely concluded that

by ohthatpatrick Mon Jan 20, 2014 12:42 am

Yeah it is really rare.

It's essentially the same task as Sufficient Assumption, because the question stem is essentially asking, "Which of these answers would prove the conclusion?"

But what's rare is that there is no premise to go off of. Normally, Sufficient Assumption would give us an argument with Prem & Conc, and we would look for an answer choice that we could add to the Prem(s) in order to prove the conclusion.

Here, the answer choice has to prove the conclusion all on its own.